Foro de preguntas y respuestas de matemáticas, de cualquier nivel. Cuánto más interesantes, divertidas o intrépidas, mejor.
Aviso: Te invitamos a conocer la página de Facebook de la UCIM

Ganas puntos al hacer preguntas, contestarlas y, sobre todo, si tu respuesta es seleccionada como la mejor.
Registrate como usuario para participar en el foro. También puedes utilizar tu identidad de FB Utiliza el botón azul para ingresar (si usas tu identidad de FB y estás logeado en FB, automáticamente te reconoce).

El irracional tiene una página en FB. El Irracional






+2 votos
Sean $A=\sqrt{2}^{\sqrt{2}^{\sqrt{2}^{.^{.^{.}}}}}$, $B=\sqrt[3]{3}^{\sqrt[3]{3}^{\sqrt[3]{3}^{.^{.^{.}}}}}$ y $C=\sqrt[4]{4}^{\sqrt[4]{4}^{\sqrt[4]{4}^{.^{.^{.}}}}}$. Se quiere ordenar de forma creciente las expresiones $A$, $B$ y $C$ en el caso de que esten definidos las convergencias correspondientes.
por (6,2m puntos) en Problemas
reetiquetada por

1 Respuesta

+4 votos
 
Mejor respuesta

Antes de responder la pregunta hacemos notar unas observaciones:

  • La expresión $x^{x^{x^{.^{.^{.}}}}}$ se denomina tetración, la cual es una operación iterada de la exponenciación, ver http://es.wikipedia.org/wiki/Tetraci%C3%B 3n.
  • La tetración no es una operación asociativa así que debe entenderse como x^(x^(x^...)).
  • La $n$-ésima iteración de la exponenciación de $x$ se denota como $^{n}x$ y se define como: $1$ si $n=0$ y $x^{[^{n-1}x]}$ si $n>0$.
  • Esta pregunta tiene una relación estrecha con la planteada en el siguiente link: http://irracional.org/index.php/67/cuando-pasa-que-%24a-b-b-a%24-para-%24a-b%24.
  • Como $4^{\frac{1}{4}}=4^{\frac{1}{2}\frac{1}{2}}=(4^{\frac{1}{2}})^{\frac{1}{2}}=2^{\frac{1}{2}}$ entonces claramente $A=C$.

Ahora bien, vamos a definir la sucesión $S=\{ ^{n}x \}$ para $x=a^{\frac{1}{a}}$ con $a\geq3$.

Afirmación 1. $S$ es creciente.

Prueba. Por inducción sobre $n$. Como $a\geq3$, entonces $0\leq \frac{1}{a}\ln{a}$, luego $1\leq x$ y por tanto $^{0}x \leq {^{1}x}$. Supongamos que $^{n-1}x\leq{^{n}x}$. Como $0\leq\ln{x}$ tenemos que $^{n-1}x\ln{x}\leq{^{n}x\ln{x}}$, así se deduce que $\ln{x^{[^{n-1}x]}}\leq\ln{x^{[^{n}x]}}$ y por lo tanto $^{n}x\leq{^{n+1}x}$. qed.

Afirmación 2. $S$ es acotada.

Prueba. Por inducción sobre $n$. Como $a\geq3$, entonces $\ln{a}\leq a$, luego $ \ln{a^{\frac{1}{a}}}\leq 1$ y por tanto $x \leq e $. Supongamos que $^{n-1}x \leq e$. Como observamos en http://irracional.org/index.php/67/cuando-pasa-que-%24a-b-b-a%24-para-%24a-b%24, la función $f(y)=\frac{y}{\ln{y}}$ tiene un mínimo en $e$, entonces $e\leq \frac{a}{\ln{a}}=\frac{1}{\frac{1}{a}\ln{a}}=\frac{1}{\ln{x}}$, así  $^{n-1}x \leq e \leq \frac{1}{\ln{x}}$, por tanto $^{n-1}x \ln{x} \leq 1$ y finalmente $x^{^{n-1}x}\leq e$. qed.

Como $S$ es creciente y acotada se concluye que converge.

Como $f(y)=\frac{y}{\ln{y}}$ tiene un mínimo en $e$, es creciente para $y\geq 3$, luego, si $3 \leq b \leq c$ tenemos que $\frac{b}{\ln{b}}\leq \frac{c}{\ln{c}}$, entonces $c^b \leq b^c$ y tenemos que $c^{\frac{1}{c}}\leq b^{\frac{1}{b}}$, por tanto si $x_b=b^{\frac{1}{b}}$ y $x_c=c^{\frac{1}{c}}$ se concluye que $^{n}x_c \leq{^{n}x_b}$. Por tal razón $C\leq B$.

También puedes ver http://luckytoilet.wordpress.com/2010/03/13/notes-on-infinite-tetration/.

por (6,2m puntos)
editado por
¡Excelente, amigos! Pronto os comparto algunas variantes, en forma de preguntas.
Licencia Creative Commons
Este obra está bajo una Licencia Creative Commons Atribución-NoComercial-CompartirIgual 2.5 México.

powered by UCIM  -  Aviso de privacidad

...